What is the measure of circumscribed LX?

O 45°
O 50°
O 90°
O 950

What Is The Measure Of Circumscribed LX?O 45O 50O 90O 950

Answers

Answer 1

Answer:

90°

Step-by-step explanation:

The angle a tangent makes with a radius at the point of tangency is 90 deg.

There are three 90-deg angles in the quadrilateral, so the 4th angle must also measure 90 deg.

Answer: 90°

Answer 2

Based on the tangent theorem the measure of the circumscribed ∠X is: 90°.

What is the Tangent Theorem?

The tangent theorem states that an angle of 90 degrees is formed at the point of tangency where a tangent meets the radius of a circle.

YX and WX are tangents of the circle.

m∠Y = m∠W = 90°

Sum of interior angles of a quadrilateral is 360°

m∠X = 360 - 90 - 90 - 90

m∠X = 90°

Therefore, based on the tangent theorem the measure of the circumscribed ∠X is: 90°.

Learn more about the tangent theorem on:

https://brainly.com/question/9892082


Related Questions

The base of a right triangle is increasing at a rate of 2 meters per hour and the height is decreasing at a rate of 3 meters per hour. When the base is 9 meters and the height is 22 meters, then how fast is the HYPOTENUSE changing

Answers

Answer:

dL/dt  = - 2,019 m/h

Step-by-step explanation:

L²  =  x²  +  y²   (1)    Where  x, and  y are the legs of the right triangle and L the hypotenuse

If the base of the triangle, let´s call x is increasing at the rate of 2 m/h

then  dx/dt =  2 m/h. And the height is decreasing at the rate of 3 m/h or dy/dt = - 3 m/h

If we take differentials on both sides of the equation (1)

2*L*dL/dt = 2*x*dx/dt  + 2*y*dy/dt

L*dL/dt  = x*dx/dt + y*dy/dt         (2)

When  the base is 9  and the height is 22 according to equation (1) the hypotenuse is:

L = √ (9)² + (22)²       ⇒    L = √565       ⇒  L  = 23,77

Therefore we got all the information to get dL/dt .

L*dL/dt  = x*dx/dt + y*dy/dt

23,77 * dL/dt  = 9*2 + 22* ( - 3)

dL/dt  =  ( 18 - 66 ) / 23,77

dL/dt  = - 2,019 m/h

Using implicit differentiation and the Pythagorean Theorem, it is found that the hypotenuse is changing at a rate of -2.02 meters per hour.

The Pythagorean Theorem states that the square of the hypotenuse h is the sum of the squares of the base x and of the height h, hence:

[tex]h^2 = x^2 + y^2[/tex]

In this problem, [tex]x = 9, y = 22[/tex], hence, the hypotenuse is:

[tex]h^2 = 9^2 + 22^2[/tex]

[tex]h = \sqrt{9^2 + 22^2}[/tex]

[tex]h = 23.77[/tex]

Applying implicit differentiation, the rate of change is given by:

[tex]2h\frac{dh}{dt} = 2x\frac{dx}{dt} + 2y\frac{dy}{dt}[/tex]

Simplifying by 2:

[tex]h\frac{dh}{dt} = x\frac{dx}{dt} + y\frac{dy}{dt}[/tex]

The rates of change given are: [tex]\frac{dx}{dt} = 2, \frac{dy}{dt} = -3[/tex].

We want to find [tex]\frac{dh}{dt}[/tex], hence:

[tex]h\frac{dh}{dt} = x\frac{dx}{dt} + y\frac{dy}{dt}[/tex]

[tex]23.77\frac{dh}{dt} = 9(2) + 22(-3)[/tex]

[tex]\frac{dh}{dt} = \frac{18 - 66}{23.77}[/tex]

[tex]\frac{dh}{dt} = -2.02[/tex]

The hypotenuse is changing at a rate of -2.02 meters per hour.

A similar problem is given at https://brainly.com/question/19954153

Which is a factor pair of 72? 1. 12, 6 2. 14, 5 3. 23, 4. 24, 2

Answers

Answer:

1. 12, 6

Step-by-step explanation:

Your answer is 1. because you need to multiply them together to get the answer. In this case all you do is 12 × 6 = 72

Hope this helps!

2√63 what is the radical

Answers

Answer:

7.94

Step-by-step explanation:

Answer:

[tex]\Large \boxed{\sqrt{63}}[/tex]

Step-by-step explanation:

A radical is the root of a number.

[tex]2\sqrt{63}[/tex]

The radical in this expression is [tex]\sqrt{63}[/tex].

Ncluding a 6% sales tax, a new stereo costs $492.9. Find the cost of the stereo before tax. A) First write an equation you can use to answer this question. Use x x as your variable and express any percents in decimal form in the equation. (1)

Answers

Answer:

1.06x = 429.9

Cost of stereo before sales tax = $405.6

Step-by-step explanation:

Given the following :

Full cost of stereo(cost after sales tax) :

(cost before sales tax + sales tax)

Sales tax = 6%

Cost after sales tax = $492.9

Take the cost before sales tax as 'x'

Therefore, cost after sales tax:

x + 6% of x = $492.9

Equation to solve the problem :

x + 0.06x = 429.9

1.06x = 429.9 - - - (1)

We can then solve for x:

1.06x = 429.9

x = 429.9 / 1.06

x = $405.56603

x = $405.6

1. Classify the following numbers as rational or irrational.
a) √45 b) √55 c) √196 d) √576 e) √27

Can u guys answer this question pls

Answers

Answer:

a) irrational

b) irrational

c) rational

d) rational

e) irrational

Note:

rational numbers are numbers that can be expressed in fractions.

irrational numbers are numbers that cannot be expressed in fractions i.e. they are never-ending-non-repeating decimals. examples √2, pi etc.


17. Test Practice which represents the number that is ten more than two
thousand, seventy?
A 280
® 2,008
© 2,080
0 2,800

Answers

Answer:

2,080

Step-by-step explanation:

Two thousand and seventy is written as 2,070.

Ten more than 2,070 would be 2,080 as you're just adding on 10.

Hope this helps :)

Find the square root of 7250 by prime factorisation.

Answers

Answer:

The square root of 7250 is 85.15

Step-by-step explanation:

[tex]{ \boxed{ \sf{7250}}} \\ ↓ \div 2 \\ { \boxed{ \sf{3625}}} \\ ↓ \div 5 \\ { \boxed{ \sf{725}}} \\ ↓ \div 5 \\ { \boxed{ \sf{145}}} \\ ↓ \div 5 \\ { \boxed{ \sf{29}}}[/tex]

Find square roots of divisors:

[tex]{ \sf{ = \sqrt{2} \times \sqrt{5 \times 5 \times 5} \times \sqrt{29} }} \\ = { \sf{ \sqrt{2} \times \sqrt{125} \times \sqrt{29} }} \\ = { \sf{85.15}}[/tex]

[tex]{ \underline{ \sf{ \blue{christ \:† \: alone }}}}[/tex]

What is the difference between {2,3} and {{2,3}}

Answers

[tex]\{2,3\}[/tex] is a set containing two elements - numbers 2 and 3

[tex]\{\{2,3\}\}[/tex] is a set containing one element - a set [tex]\{2,3\}[/tex]

find the product of the first 3 positive integers and then the first 5 negative integers.

Answers

Answer:

6 and -120

Step-by-step explanation:

The first 3 positive integers are 1, 2 and 3 and their product is 6, the first 5 negative integers are -1, -2, -3, -4 and -5 and their product is -120.

Find the missing probability: P(B)=7/20, P(A|B)=1/4, P(A∩B)=?

Answers

Answer:

P(A∩B) = 7/80

P(A∩B) = 0.0875

Step-by-step explanation:

Given

P(B)=7/20

P(A|B)=¼

Required

P(A∩B)=?

The given probability shows conditional probability and the relationship between the given parameters is as follows.

P(A∩B) = P(B) * P(A|B)

Substitute ¼ for P(A|B) and 7/20 for P(B)

The expression

P(A∩B) = P(B) * P(A|B) becomes

P(A∩B) = 7/20 * ¼

P(A∩B) = 7/80

P(A∩B) = 0.0875

Hence, the calculated P(A∩B) is 7/80 or 0.0875

If 1 angle is four time of another angle in linear pair find the angles. Please do it fast as you can

Answers

Answer:

let the angle be x then other angle is 4x

so,

x + 4x = 180

5x = 180

x = 36

so other angle I.e 4x = 4 × 36 = 144

Let one angle be x

Other angle=4x

Both are linear pair hence their sum will be 180

[tex]\\ \sf \longmapsto x+4x=180[/tex]

[tex]\\ \sf \longmapsto 5x=180[/tex]

[tex]\\ \sf \longmapsto x=\dfrac{180}{5}[/tex]

[tex]\\ \sf \longmapsto x=30[/tex]

[tex]\\ \sf \longmapsto 5x=5(30)=150[/tex]

34. Which is the rule for the linear
function graphed below?

Answers

Answer:

We can note that this part of the graph is a linear function. This means that is has a general form: y = mx + c where m is the slop and c is the y-intercept (value of y at x=0). For the slope, we will use the points (0,2) and (3,5) to calculate it as follows: m = (y2-y1)/(x2-x1) = (5-2)/(3-0) = 1 For the y-intercept, we can note that at x=0, the value of y is 2. This means that the equation of the first part of the graph is: y = x + 2

Read more at Answer.Ya.Guru – https://answer.ya.guru/questions/703068-which-rules-define-the-function-graphed-below.html

#1 why is it B?
No idea someone help plz

Answers

Answer:

Option (B)

Step-by-step explanation:

1). Given function is,

  [tex]f(x)=\frac{1}{x-1}+3[/tex]

  It the given function 'f' is transformed by a translation of 2 units to the right, the new function will be,

h(x) = f(x - 2)

h(x) = [tex]\frac{1}{(x-2)-1}+3[/tex]

      = [tex]\frac{1}{x-3}+3[/tex]

Further the new function is translated by 6 units down,

g(x) = h(x) - 6

g(x) = [tex]\frac{1}{x-3}+3-6[/tex]

      = [tex]\frac{1}{x-3}-3[/tex]

Since, transformed function 'g' passes through a point (x, -2),

g(x) = [tex]\frac{1}{x-3}-3[/tex]

-2 = [tex]\frac{1}{x-3}-3[/tex]  

3 - 2 = [tex]\frac{1}{x-3}[/tex]

x - 3 = 1

x = 4

Therefore, Option (B) will be the answer.

an inch worm is how long in general

Answers

Answer:

A inch

Step-by-step explanation:

what is the distance between (-3,0,1) and (5,2,0).​

Answers

Answer:

4

Step-by-step explanation:

Answer:

Step-by-step explanation:

[tex]d=\sqrt{(x_{2}-x_{1})^2+(y_{2}-y_{1})^2+(z_{2}-z_{1})^2}\\ =\sqrt{(5+3)^2+(2-0)^2+(0-1)^2} \\=\sqrt{64+4+1} \\=\sqrt{69}[/tex]

What is a negative rational number times a positive integer?
A) A negative integer
B) A positive rational number
C) A negative rational number
D) An irrational number

Answers

Answer:

C

Step-by-step explanation:

The product of a negative rational number and a positive integer is a negative rational number

Answer: C (A Negative Rational Number)

Step-by-step explanation

A Negative number mutiplied by a positive number is a negative number.

Please click thanks and mark brainliest if you like

If I Have 100 Dollars And i go to the bank and i ask for 30 more and use that money on a store and i have 12 Dollars Left How Much Did the Thing I Bought Cost/ How Much Money did i spend?

Answers

Answer:

$118

Step-by-step explanation:

First, find how much money you had after going to the bank:

100 + 30

= $130

Find how much money you spent at the store by subtracting 12 from 130:

130 - 12

= 118

So, you spent $118

Can someone help?...look at the pics

Answers

Answer:

[tex]\boxed{y=2x-2}[/tex]

Step-by-step explanation:

Pick values from the table.

When x = 1, y = 0.

The third option seems right.

[tex]y=2(1)-2[/tex]

[tex]y=2-2[/tex]

[tex]y=0[/tex]

True.

Find all positive integers less than 100 whose 762nd power ends in the digits "41"

Answers

Answer:

  21, 29, 71, 79

Step-by-step explanation:

Only integers ending in 1, 3, 7, or 9 have powers that end in 1. For the numbers so ending, their sequences of powers mod 100 have a repeat length that is a divisor of 20. That is, raising any of these numbers to the 762nd power mod 100 is equivalent to squaring them, mod 100.

For example, the powers of 29 mod 100 are, beginning with the first power, ...

  {29, 41, 89, 81, 49, 21, 9, 61, 69, 1}

This sequence is of length 10, typical of many of the numbers ending in 1, 3, 7, or 9.

The only positive integers less than 100 whose squares end in 41 are ...

  21, 29, 71, 79 . . . . . positive integers < 100 satisfying n^762 mod 100 = 41

PLS ANSWER I WILL GIVE BRAINLIST AND A THANK YOU

Answers

Answer:

x= 6 degrees

Step-by-step explanation:

x+x+54 = 90

2x+54 = 90

x= 90- 54 /2

x =6

Answer:

x = 6

Step-by-step explanation:

90 - 54 = 36

x + 5x = 6x

36 + 6x = 90

90 - 36 = 6x

36/6x = 6

x = 6

Given the graph of a radical function, which statement is correct?

Radical function going from the point negative 4 comma negative 3 up to the right through the point 5 comma 0

R colon open bracket y is an element of all real numbers close bracket
R colon open bracket y is an element of all real numbers such that y is greater than or equal to 5 close bracket
R colon open bracket y is an element of all real numbers such that y is greater than or equal to negative 4 close bracket
R colon open bracket y is an element of all real numbers such that y is greater than or equal to negative 3 close bracket

Answers

Answer:

R colon open bracket y is an element of all real numbers such that y is greater than or equal to negative 3 close bracket

1. ABCD is a quadrilateral in which angle A = angle C and angle B = angle D
Prove that, if the opposite angles of a quadrilateral are equal, then the quadrilateral is a
parallelogram.

Answers

Answer:

Quadrilateral ABCD is a parallelogram, quadrilateral with opposite sides parallel

Step-by-step explanation:

Statement,                                       Reason

∠A = ∠C,                                          Given

∠B = ∠D,                                          Given

Therefore, we have

∠A + ∠B + ∠C + ∠D = 360°,            Sum of interior angles of a quadrilateral

2×∠A + 2×∠B = 360°,                         Substitution property of equality

∴ ∠A + ∠B = 360°/2 = 180°

∠A + ∠B, are supplementary  angles,   Angles that sum up to 180°

Similarly,

∠C + ∠D, are supplementary  angles,   Angles that sum up to 180°

Therefore, segment DA is parallel to segment BC, (Lines with supplementary angles on the same side of the transversal)

Similarly, given ∠A = ∠C and ∠B = ∠D, we have;

∠A + ∠D and ∠C + ∠B are supplementary  angles,   Angles that sum up to 180°

Therefore, segment AB is parallel to segment DC, (Lines with supplementary angles on the same side of the transversal)

Therefore;

Quadrilateral ABCD is a parallelogram, quadrilateral with parallel opposite sides.

by selling an article sonu makes a profit of 20%. if the cp decreased by 10% and sp also increased by 10%,calculate her profit percentage​

Answers

Answer:

37.8 %

Step-by-step explanation:

Let CP = 100

[tex]SP =\frac{100+profit}{100}*CP\\\\=\frac{120}{100}*100[/tex]

SP = 120

New CP:

CP decreased by 10%

 [tex]Decreased \ amount=\frac{10}{100}*CP\\\\=\frac{10}{100}*100[/tex]

 = 10

New CP = 100- 10 = 90

New SP:

SP increased by 10%

Increase amount = [tex]\frac{10}{100}*old \ SP[/tex]

                            [tex]= \frac{10}{100}*120\\\\= 12[/tex]

New SP = 120 + 12 = 132

Profit = new SP - new CP

         = 132 - 90 = 42

Profit percentage = [tex]\frac{Profit}{CP}*100[/tex]

                             [tex]= \frac{42}{90}*100\\[/tex]

                             = 46.67%

Step-by-step explanation:

Here your ans..

HOPE IT HELPS YOU.....

PLEASE MARK ME BRAINLIST.....

Compare the functions shown below:

Which function has the greatest maximum y-value?

Answers

Answer:Hey I'm sorry I didn't get to answer your question it's just that I need the points because I don't have enough to get help with my question. I hope you get the answer that you need for you question. Good Luck :)

Step-by-step explanation:

45 laboures can repair a road in 24 days .how many labourers should be added to repair the road in 20 days​

Answers

Answer:

here's your answer .Hope you will understand it.

Answer:

9

Step-by-step explanation:

hope it will help good day.

Two trees are leaning on each other in the forest. One tree is 19 feet long and makes a 32° angle with the ground. The second tree is 16 feet long. What is the approximate angle, x, that the second tree makes with the ground? A 0.6° B 35.0° C 39.0° D 58.0°

Answers

Answer:

C 39.0

Step-by-step explanation:

To find the approximate angle, x, that the second tree makes with the ground, we can use the concept of similar triangles Therefore the correct option is B.

Let's calculate the height of the first tree using the given information. We can use the formula for the opposite side in a right triangle: opposite = adjacent * tan(angle). Therefore, the height of the first tree is approximately [tex]19 * tan(32°) = 19 * 0.6249 ≈ 11.873[/tex]  feet. Now, we can set up a proportion between the two trees based on their heights. Let x be the angle the second tree makes with the ground.

We have the following proportion: (height of first tree)/(height of second tree) = (length of first tree)/(length of second tree). Substituting the known values, we have [tex]11.873/16 = 19/x[/tex]. Cross-multiplying gives us [tex]11.873x = 304,[/tex]  and dividing both sides by 11.873 yields[tex]x ≈ 25.63°.[/tex]  The approximate angle, x, that the second tree makes with the ground is closest to 35.0°.

Hence the correct option is B

To know more about triangle visit:

https://brainly.com/question/30966657

#SPJ2

helphelphelphelphelphelp

Answers

Answer:

see image

Step-by-step explanation:

A 3cm x 2cm rectangle sits inside a circle with a radius of 4cm
What is the area of the shaded region
round your final answer to the nearest hundredth

Answers

The area of the shaded region is 44.24\ cm^{2}44.24 cm2

Step-by-step explanation:

we know that

The area of the shaded region is equal to the area of the circle minus the area of rectangle

step 1

Find the area of circle

The area of the circle is equal to

A=\pi r^{2}A=πr2

we have

r=4\ cmr=4 cm

substitute

A=\pi (4)^{2}A=π(4)2

A=16\pi\ cm^{2}A=16π cm2

step 2

Find the area of rectangle

The area of rectangle is equal to

A=(3)(2)=6\ cm^{2}A=(3)(2)=6 cm2

step 3

Find the difference

16\pi\ cm^{2}-6\ cm^{2}16π cm2−6 cm2

assume

\pi=3.14π=3.14

16(3.14)\ cm^{2}-6\ cm^{2}=44.24\ cm^{2}16(3.14) cm2−6 cm2=44.24 cm2

Select the equivalent expression.
(x^-3*y^3)^-7=?
Choose 1 answer:

Answers

Answer:

The equivalent expression for this equation is x²¹ × y⁻²¹

Step-by-step explanation:

When you are multiplying exponents that are in parentheses, you multiply the exponential numbers together. So, for this problem, we will multiply the exponents together so we can get our final equivalent expression.

(x⁻³ × y³)⁻⁷

So, we will multiply -3 by -7 and we will also multiply 3 by -7 because those are our exponential numbers.

x²¹ × y⁻²¹

So, this is the final expression that is equivalent to our equation.

The expression (x⁻³.y³)⁻⁷ is equivalent to x ²¹ . y ⁻²¹.

What is algebra?

Algebra is a study of mathematical expressions, in which numbers and quantities are represented in formulas and equations by letters and other universal symbols.

The given expression is,

(x⁻³.y³)⁻⁷

To find the solution for the expression,

Simplify the power terms,

x⁽⁻³ˣ⁻⁷⁾ . y ³ˣ⁻⁷

x ²¹ . y ⁻²¹

The simplified form of the expression (x⁻³.y³)⁻⁷ is x ²¹ . y ⁻²¹.

Hence, option (A) is correct.

To know more about Algebra on:

https://brainly.com/question/24875240

#SPJ2

a 6 foot tall man casts a shadow that is 9 ft long. At the same time, a tree nearby casts a 48 ft shadow. how tall is the tree

Answers

Answer:

32 ft tall

Step-by-step explanation:

Since a 6 ft man casts a shadow 9 ft long, the shadow is 3/2 of the actual object/person.

SINCE THE TREE'S SHADOW IS AT THE SAME TIME, THE HEIGHT IS THE SAME RULE.

We know the tree's shadow is 48 ft.

--> 48/3 = 16

16 x 2 = 32

32 ft tall

Hope this helps!

Answer: 32ft tall

Step-by-step explanation:

Other Questions
A substance is followed by the symbol (I) in a chemical Equation. What does the symbol Represent how did the Louisiana Purchase affect women rights Richard is a licensed agent who represents Spartan Health Plan and its Medicare Advantage (MA) plans. Richard has several clients who have recently come to him for help who are in their initial coverage election period (ICEP) and are interested in enrolling in one of Spartan Health Plans MA plans. Alice will soon turn 65 and retire. Alice has coverage through Spartan Health Plan offered by her employer. Bob had health coverage through Spartan but dropped the coverage when he retired early to travel overseas. Bob, who has just turned age 65, is now back in the United States. Charlotte, who will turn 65 next month, has coverage through Athena Health plan a company Richard also represents. Who qualifies for the opt-in simplified enrollment mechanism? How do you think these two disciplines lend themselves to one another? Given that log 2 = 0.3010 and log 7 = 0.8451, find the following (a) log 49 (b) log 560 The price of a technology stock was $ 9.56 yesterday. Today, the price rose to $ 9.69 . Find the percentage increase. Round your answer to the nearest tenth of a percent. Explain how the atomic theory was modified from Daltons theory to Rutherfords. Include an explanation of why the theory was modified Patios can be made by mixing cubic meters of ash, stone, and wood chips in the ratio 5:7:3. How much stone is needed to make 45 cubic meters of patio? Robert says that his friend Joseph reads a lot of poetry and also exercises for long hours at his local fitness center. Robert asks his friend Grace whether Joseph is more likely to be an English professor or a fitness trainer. Grace replies that Joseph is most likely both. She says that Joseph cannot only be a professor or fitness trainer. This is an example of:A. the bandwagon effect.B. the focusing effect.C. egocentric bias.D. the conjunction fallacy. Smathers Corp. stock has a beta of 1.23. The market risk premium is 7.00 percent and the risk-free rate is 2.86 percent annually. What is the company's cost of equity? Which correlation best describes the data below.no correlationweak positivestrong positivestrong negative Consider the following Causal Claim:All of a sudden he's all Let's go to Beano's for a change. Right. Am I supposed to think it's just coincidence his old girlfriend started working there?This is an example of:_______.A. Common Thread reasoning. B. Relevant Difference reasoning. Role of mitosis in plant growth and development Select all the correct answers.Which two themes are present in the poem?Our future is often determined by fate.Roads are the best way to travel.Life presents many choices for us to make.Life presents many unfair challenges for us.Our future depends on the choices we make. Sanchez Inc. reported the following on the company's cash flow statement for 20Y7: Net cash flow from operating activities$500,000 Net cash flow used for investing activities(200,000) Net cash flow used for financing activities150,000 Thirty percent of the cash flow used for investing activities was used to purchase property, plant, and equipment. What is the free cash flow for 20Y7? mitosis cell division are required to form 64 cell for one cell?a.4b.6c.16d.32 QuWhat can you determine about the feasibility of areaction if the enthalpy is positive and the entropy isnegative?1A. The reaction will usually occur because it is unlikely theentropy will be greater than the enthalpy.B. The Gibbs energy will always be positive, and the reaction willnever be feasible.C. The Gibbs energy will always be negative, and the reaction willalways be feasible.D. The reaction could be feasible above a certain temperature,Reset Selection Hsung Company accumulates the following data concerning a proposed capital investment: cash cost $175, 846, net annual cash flows $37, 300, and present value factor of cash inflows for 10 years 5.02 (rounded). (If the net present value is negative, use either a negative sign preceding the number eg -45 or parentheses eg (45).) Determine the net present value, and indicate whether the investment should be made? Sam is a young adult who is enrolled in college in the United States. He is taking a broad range of courses, and is passionate about automobiles and motorcycles. He has the capacity to perform tasks that Piaget would deem formal operational; however, he selectively uses his cognitive ability. In which of the following courses is he more likely to perform cognitive tasks that are within formal operations?a. literatureb. small engine repairc. computer-aided designd. psychology and sociology Find the work done by the force field F(x,y,z)=6xi+6yj+6k on a particle that moves along the helix r(t)=3 cos(t)i+3sin(t)j+2 tk,0t2.